Recent content by Dumbledore

  1. D

    Calculating Final Rotation of Spinning Wheel

    I used AT's links there to solve for t, but I am still a little stuck because this seems to be a far more complex problem than I originally realized. The current rotation is updated with the angular velocity multiplied by dt. However, it is based on framerate. So if the function is called...
  2. D

    Calculating Final Rotation of Spinning Wheel

    @A.T. thanks, that was the information I was looking for. @Doc Al This isn't a homework assignment where there is a "problem" description. I have created this problem by choosing this method of calculating change in angular velocity for a little game I am writing. I would like to hear...
  3. D

    Calculating Final Rotation of Spinning Wheel

    Hey guys, I guess I'm even worse than I thought at this stuff :( So I used coefficient of friction incorrectly. Basically the equation I have there is all the information available. The equation is essentially saying: update my current speed by subtracting my current speed with the...
  4. D

    Calculating Final Rotation of Spinning Wheel

    My understanding is that V = velocity in radians. 0.185 per second time in seconds So the seconds cancel each other out. (Cause it is really 0.185/1sec) That leaves you with a number representing angular velocity in radians which is the radians per second... I don't know much about...
  5. D

    Calculating Final Rotation of Spinning Wheel

    The equation is like this: Vnext = Vcurr - (Vcurr * 0.185 * elapsedTimeInSeconds) Practical examples given initial velocity (V1) at 10, with a dT of 0.5 (every 0.5 seconds, the velocity gets updated) V2 = 10 - (10 * 0.185 * 0.5) = 9.075 V3 = V2 - (V2 * 0.185 * 0.5) = ?? Etc... Now I'm...
  6. D

    Calculating Final Rotation of Spinning Wheel

    Hello, I am really struggling with calculating the final rotation of a spinning wheel. I am not even sure if my equation for changing the velocity of the spinning wheel given a coefficient of friction is even correct so I will post it here... change in angular velocity = angular velocity *...
  7. D

    Differentials within integrals

    Yeah you are right. Basically, everything I said is completely incorrect. This entire thread is an embarrassment.
  8. D

    Differentials within integrals

    I'm pretty sure that is incorrect, but I don't have the math background to know for sure. I see that I did make yet another mistake though... I'll show you how you can solve this without integrating by parts: Integral( ln(2x) ) = 1/2 Integral ( ln(2x) 2dx) = 1/2 (1/2x) (2) = 1/2x Is this not...
  9. D

    Differentials within integrals

    I think I get it, its because the differential of x is dx, and the differential of u is du. So if you have a function of a function you have to identify u and find du. In ln x u = x du = dx So in this case I am incorrect to say Integral (ln x) == Integral ( ln x 1/x dx) But if it were ln...
  10. D

    Simple integral, textbook seems wrong

    Homework Statement Find the equation of the curve for which the slope is (ln x)^2/x and passes through P(1, 2) Homework Equations The Attempt at a Solution Integrate (ln x)^2 = 1/2 Integral( ((ln x)^2) 2/x dx) I get: 1/2 [((ln x)^3/3) + C] Then solving for C, I get C=2...
  11. D

    Differentials within integrals

    Hello. Can someone please explain why I have to transform an integral of a differential function into the form Integral ( lnx 1/x dx ) for example, for Integral ( lnx ). It seems to only be done with transcendental functions and not the algebraic ones... ie. Integral ( x^2 ) != Integral...
  12. D

    Simple differentiatian of tan question

    Homework Statement What is the time rate of change of height after 5.0s?Homework Equations theta = 3t/(2t + 10) adjacent length = 1000 Therefore: opposite length (height) h = 1000 tan (3t/(2t+10)) The Attempt at a Solution I thought it would be simply finding the derivative of the second...
  13. D

    Area of simple curve bounded by

    My textbook is incorrect as I suspected. It claims the answer is 3. Thanks for your help.
  14. D

    Area of simple curve bounded by

    Thanks I finally understand why I have to use two integrals to figure out this question if I use verticle elements. For horizontal elements, I did this: Integral(3-1): (4/y^2) (dy) Integrate: -4 * y^(-1) Solve: -4*(1/3) - (-4)(1) Equals: 2 and 2/3 What did I do wrong this time?
Back
Top